Existence / non-existence d'une séquence avec une courte ultérieurement la plus longue et la sous-séquence diminue?

cs.stackexchange https://cs.stackexchange.com/questions/128260

Question

Peut exister une séquence entière $ a $ de longueur $ n $ avec tous les éléments uniquesque la longueur de sa plus longue de la recherche de plus en plus longue que celle de sa plus longue de la recherche préalable est inférieure à $ \ displaystyle \ lflfor \ frac {n} {2} \ rfloor $ ?

Si oui, donnez un exemple d'une telle séquence.Sinon, quelqu'un peut-il présenter une preuve qu'il ne peut exister qu'une telle séquence?

(juste pour ajouter une substance, peut-il être montré qu'il peut exister de telles séquences, étant donné une valeur arbitraire de $ n> 1 $ ?)

Était-ce utile?

La solution

La réponse à la question de l'OP est, non si $ n \ le 7 $ et oui sinon.


pour tout entier positif $ R $ et $ s $ , Le célèbre théorème Erdős-Szekeres montre que pour toute séquence de nombres réels distincts avec une longueur au moins $ (R-1) (S - 1) (S - 1) + 1 $ contient une ultérieurement croissante de la longueur de la longueur $ r $ < / étendue> ou une sous-séquence décroissante de la longueur $ s $ .

Il s'avère que la liaison, $ (R-1) (S-1) +1 $ est serré. C'est-à-dire que pour tout nombre positif $ r $ et $ s $ , il existe une séquence de nombres distincts avec longueur $ (R-1) (S-1) $ qui ne contient pas de nouvelle séquence de longueur de longueur $ r $ et ne pas diminuer la recherche de longueur $ s $ .

Voici un exemple aussi.

$$ \ commencez {array} {} & S-1, & S-2, & \ Cdots, & 2, et 1 \\ & 2 (S-1), & (S-1) + S-2, & \ CDOTS, & (S-1) + 2, & (S-1) + 1 \\ & \ VDOTS & \ VDOTS & \ VDOTS & \ VDOTS & \ VDOT \\ & (R-2) (S-1), & (R-3) (S-1) + S-2, & \ CDOTS, & (R-3) (S-1) +2, et (R- 3) (S-1) +1 \\ & (R-1) (S-1), & (R-2) (S-1) + S-2, & \ CDOTS, & (R-2) (S-1) +2, et (R- 2) (S-1) +1 \\ \ fin {array} $$

Considérez les chiffres ci-dessus, en lisant de gauche à droite puis de haut en bas. En d'autres termes, la séquence est $ S-1 $ jusqu'à $ 1 $ , suivi de 2 $ (S-1) $ vers le bas de $ (S-1) +1 $ , etc., et enfin suivi de < SPAN CLASS="MATH-CONTENEUR"> $ (R-1) (S-1) $ DOWN to $ (R-2) (S-1) +1 $ , tout à l'étape de 1 $ $ .

Il est facile de voir qu'il n'y a pas de multiplication de la durée de la longueur de la longueur et ne décroissant pas la recherche de longueur de longueur $ s $

.

.

Par exemple, lorsque $ R= S= 5 $ , nous avons $$ 4,3,2,1, \ \, 8,7,6,5, \ \, 12,11,10,9, \ \, 16,15,14,13 $$ qui n'a pas d'augmentation de la défense de la longueur de la longueur 5 $ ni diminution de la défense de la longueur de la longueur 5 $

.

.


Si nous laissons $ R= S $ , la section ci-dessus implique que, pour tout numéro positif $ n $ < / SPAN>, il existe une séquence entière de longueur $ n $ avec tous les éléments uniques de telle sorte que la durée de sa plus longue recherche croissante ainsi que celle de sa plus longue duréage décroissant est au plus $ \ lceil \ sqrt n \ rceil $ . Et $ \ lceil \ sqrt n \ rceil $ est la limite supérieure serrée.

depuis $$ \ lceil \ sqrt n \ rcil \ ge \ lfloor \ frac n2 \ rfloor \ \ text {pour tous} n \ le 7 $$ et $$ \ lceil \ sqrt n \ rcil \ lt \ llfloor \ frac n2 \ rfloor \ \ text {pour tous} n \ gt 7, $$ La réponse à la question de l'OP est, non si $ N \ LE 7 $ et oui sinon.

Par exemple, pour $ N= 8 $ , nous avons la séquence 3,2,1,6,5 $, 4,9,8,7 $ .

Autres conseils

Voici une construction directe d'une telle séquence pour tout multiple de quatre. Il est composé de quatre courses de taille égale de nombres entiers consécutifs.

Les première et troisième courses augmentent. Les deuxième et quatrième courses diminuent. Les exécutions utilisent des gammes de nombres tels que $ r_2 . Par exemple, avec 4N $= 16 $ ,

$$ 9,10,11,12 | 4,3,2,1 | 5,6,7,8 | 16, 15,14,13 $$

La recherche croissante la plus longue est la longueur N + 2 $ . Par exemple, dans ce qui précède où 4n $= 16 $ , la ultérieurement augmentant la plus longue a une longueur 6 $ ( 1 $ | 5, 6, 7, 8 | 16 $ ). Aucune subséquence croissante n'est plus longue:

  • Il n'est pas possible de choisir un élément à la fois d'augmenter les pistes, puisque tout élément de la première course croissante qui les disqualifie tous de la deuxième course croissante.
  • Il n'est pas possible de choisir plus d'un élément de l'une des deux lignes décroissantes

Un argument symétrique s'applique aux sous-séquences décroissantes.

depuis $ n + 2 << 2n $ , cela fonctionne comme un contre-échantillon pour une séquence multiximative de quatre. Vous pouvez facilement rembourrer avec des éléments de séquence supplémentaires pour des longueurs non multiples de quatre.

Je suis tombé sur cette construction en tenant compte d'une séquence qui était une "colline" (croissante, puis diminue), ce qui répond parfaitement à votre état. Rompre ces longues courses pourraient être faites en faisant deux collines (augmenter, diminuer, augmenter, diminuer), que cette séquence effectue en assurant la pente haut / bas d'une «colline» ne se poursuivie pas par l'autre.

Il existe également des séquences courtes qui satisfont à votre demande. Considérez par exemple les 16 premiers termes de la séquence Binary Van der Corput $$ 0, 8, 4, 12, 2, 10, 6, 14, 1, 9, 5, 13, 3, 11, 7, 15. $$ En général, il existe une séquence $ t $ de longueur $ n \ geq1 $ contenant une ultérieurement croissante delongueur $ x \ geq 1 $ et une ultérieurement décroissante la plus longue de longueur $ y \ geq 1 $ si etSeulement si les chiffres $ x $ , $ y $ et $ N $ satisfaire les conditions $ x \ cdot y \ geq n $ et $ x + y \ leqn + 1 $ , voir ici .Notez que la référence donne une preuve constructive.

Ces séquences existent.Il suffit de générer une séquence aléatoire suffisamment grande.Si vous consultez le livre de Dan Romik, Les mathématiques surprenantes des ultérieurements croissants , théorème 1.1 indique que

$$ \ frac {\ ell_n} {\ sqrt n} \ to 2, $$

$ \ ell_n $ est une longueur attendue de la défense croissante dans une permutation aléatoire de la taille $ n $ .La même chose pour diminuer.Par conséquent, pour suffisant assez $ n $ Il doit exister une séquence avec des séquences croissantes et décroissantes de longueurs au plus $ 5 \ sqrtn $ , sinon:

$$ 2 e [\ ell_n]= e [| DREN_N |+ | incr_n |] \ ge 5 \ sqrt n, $$

qui contredit le théorème.

Licencié sous: CC-BY-SA avec attribution
Non affilié à cs.stackexchange
scroll top